LSAT and Law School Admissions Forum

Get expert LSAT preparation and law school admissions advice from PowerScore Test Preparation.

User avatar
 Dave Killoran
PowerScore Staff
  • PowerScore Staff
  • Posts: 5853
  • Joined: Mar 25, 2011
|
#49518
Setup and Rule Diagram Explanation

This is a Grouping Game: Defined-Fixed, Balanced.

S95_Game_#1_setup_diagram 1.png
The combination of the second, third, and fifth rules leads to the following inferences involving W
and Y:
  • ..... ..... ..... ..... ..... ..... ..... W1 :arrow: Y2

    ..... ..... ..... ..... ..... ..... ..... W2 :arrow: Y1
The application of the final rule leads to a single solution:

S95_Game_#1_setup_diagram 2.png
Because this rule is so limiting, you should expect it to be tested at most once or twice during the game.
You do not have the required permissions to view the files attached to this post.
 LSAT2018
  • Posts: 242
  • Joined: Jan 10, 2018
|
#46342
There did not seem to be as much inferences that could be made, so do you mind looking over these?

Given that S must be added to class 3, Y cannot be in class 3.
Given the final rule 'If T is added to class 1, Z must also be added to class 1' I found that once that conditional statement is in effect the whole diagram is filled.
1: R, T, Z
2: Y, X, V
3: W, S

After solving the questions, I found that alot of the answers involved X. Am I missing an inference on X?
Last edited by LSAT2018 on Sat Jun 16, 2018 2:58 am, edited 1 time in total.
User avatar
 Dave Killoran
PowerScore Staff
  • PowerScore Staff
  • Posts: 5853
  • Joined: Mar 25, 2011
|
#46644
Hi LSAT2018,

This game is one of my all-time favorites :) It displays the Hurdle the Uncertainty concept perhaps better than any other I've ever seen, especially in questions #3, 4, and 5, so that's the biggest takeaway. It also means that the setup isn't full of killer inferences.

With Y knocked out of class 3, and also not sharing a class with W, we can make the following inference:

..... ..... W1 :arrow: Y2
..... ..... W2 :arrow: Y1

You are correct about the final rule yielding only one solution, so don't expect to see it often in the game.

The real focus in this game is on the VZ and WY not-blocks. Those blocks dominate the questions mostly, and it's why X keeps coming up so often: even though X is a random, it often ends up being forced into a certain class as the not-blocks take up spaces in each class (even though you don't know which variable it is—the essence of Hurdle the Uncertainty).

Please let me know if that helps. Thanks!
User avatar
 Sjones
  • Posts: 1
  • Joined: Dec 08, 2022
|
#98515
HI, I am wondering how there can be only one solution.
Could would not have for instance
1 - RYZ
2 TXV
3 SW
 Adam Tyson
PowerScore Staff
  • PowerScore Staff
  • Posts: 5153
  • Joined: Apr 14, 2011
|
#98516
That reference to "one solution" in the explanation is just about what happens when the final rule of the game is activated by putting T in the first class. There is only one solution IF T is in Class 1. But there are, of course, many solutions with T somewhere else! You found one, but there are many others.

Get the most out of your LSAT Prep Plus subscription.

Analyze and track your performance with our Testing and Analytics Package.